Download as pdf or txt
Download as pdf or txt
You are on page 1of 15

DAY-08

DAY-26
DAY 08

DAY 01

1
DAY-08

Dear Students,
I hope this message finds you well and motivated for your upcoming Bank Mains Exams. As you
prepare to tackle this significant challenge, I want to offer you some words of encouragement and
guidance to help you succeed.
1. Stay Committed: Remember why you started this journey. Keep that motivation alive and use it
to fuel your dedication towards your goal. The journey might be tough, but your determination
will see you through.
2. Plan Wisely: Create a detailed study plan that covers all the subjects and topics you need to study.
Allocate time wisely, giving more attention to your weaker areas while also revising your strengths.
3. Practice Regularly: Consistent practice is key to success. Rank File is a collection of most effective
and exam related questions. This will help you manage your time effectively during the exam.
4. Stay Healthy: Good physical and mental health is crucial for effective preparation. Eat well,
exercise, and take breaks to recharge. Don't compromise your well-being for the sake of studying.
5. Stay Positive: Maintain a positive attitude, even when facing difficult concepts or a challenging
mock test. A positive mindset will help you overcome obstacles more easily.
6. Time Management: Learn to manage your time during the exam. Don't get stuck on a single
question; move on and return to it later if necessary. Allocate time wisely to different sections.
7. Believe in Yourself: You've put in the effort, and you are capable. Believe in your abilities and stay
confident throughout the exam.
Remember, success is not just about the destination but also about the journey. Embrace the learning
process, and every challenge you face will be a stepping stone towards your goal. Keep your eyes on
the prize, and you will surely excel in your Bank Mains Exam.
Wishing you the best of luck and success!
[SHANTANU SHUKLA]
Sr. Educator, Adda 247

2
DAY-08

DAY 08
S. No. Topic Sub Topic No. of Questions
1 Arithmetic With Variable 9
2 Number series Wrong 4
3 Quantity Comparison 2 &3 statements 5
4 Data Interpretation Caselet (Cricket) 5
5 Data Interpretation Caselet 3
6 Data Interpretation Table (Based on 5
point)
7 Data Interpretation Line graph + Table 5
8 Approximation 4
Total 40

3
DAY-08
Directions [Q1-5]: with respect to previous match given that the
Answer the questions based on the information opposition (Mumbai) conceded 4 extra runs
given below. in this match?
In 4 match T20 series played between Delhi and (a) 10% (b) 12%
Mumbai, only 3 Delhi players namely Hardik, Rahul (c) 15% (d) 20%
and Kaif got to bat. Ratio of runs scored by Hardik in (e) None of these
match1, Rahul in match 2 and Hardik in match 3 was
4:5:6, respectively. Runs scored by Rahul in match 2 Direction (Q6-10):
were (2/3)rd of runs scored by him in match 1, Read the following information carefully and answer
whereas he scored twice as much runs in match 3 as the questions based on it.
he did in match '2'. In match 2, Rahul scored 25% There are four persons A, B, C and D started
more runs than Hardik and 20% less runs than Kaif. travelling from Delhi at same time and reached
Overall, Rahul scored 125 runs more than Kaif. Runs Punjab by their own bikes with same route. In overall
journey they all are comparing their driving skills
scored by Kaif in match 1, Rahul in match 2 and Kaif
and certain points are awarded to them.
in match 3 were in ratio 3:5:7, respectively.
Name Number Time taken Number
of WL to reach of SL
1. Find the total score of Delhi in match 2, given
Punjab
that there were no extra runs conceded by
Mumbai team. ( in hours)
(a) 310 (b) 320 A 5 8 3
(c) 305 (d) 290 B I 6 H
(e) 460 C F 12 G
D 8 4 3
2. Find the difference between runs scored by Points earned = 20 x number of wheelies (WL) – 8 x
Rahul in match 3 and runs scored by Hardik number of slips (SL)
and Kaif together, in match '1'. Note: No person scored negative points and each
(a) 80 (b) 60
person has done at least one wheelie and one slip.
(c) 120 (d) 70
(e) 90 6. Points earned by B are 52 less than that of D
and number of slips done by B is not more
3. Find the respective ratio of runs scored by than number of wheelies done by him, and
Rahul in match 1 and runs scored by Kaif in then find which of the following can be the
match 1 and 3, together.
possible value of
(a) 4:3 (b) 3:1
(2I + 3H)?
(c) 3:2 (d) 3:4
I. 35
(e) None of these
II. 54
4. If Mumbai lost match 3 by 79 runs, then find III. 16
the runs scored by Mumbai batsmen in match (a) III only (b) II only
3, given that Delhi conceded 31 extra runs (c) I and III only (d) II and III only
while Mumbai was batting. (e) None of these
(a) 381 (b) 391
7. Points scored by B is (K – 4), where K is the
(c) 387 (d) 250
least composite number, then find the
(e) None of these
minimum positive possible points earned by
5. If in match 4 played by Delhi, Rahul scored C, if number of slips done by C is twice as that
25% less runs than he did in match 3, of B?
whereas Hardik and Kaif scored same runs, (a) 40 (b) 16
as they did in the match '3'. By how much (c) 24 (d) 20
percent, did the total score of Delhi team drop (e) None of these

4
DAY-08
8. Points earned by C are 24 more than that of A, Notes:
then find the minimum possible value of (F + Total quantity of potatoes sold = 400 kg
G)? P does not go to house M
(a) 12 (b) 9 T does not go to house M and O.
(c) 16 (d) 18 Q went to house M and N only.
(e) None of these The number of times did S go to house M is 4 more
than the number of times did S go to house O
9. Points earned by B are less than D, and then
find maximum possible value of I and 11. R went all 3 houses. R went to house M for 7
minimum possible H and then find (I + H)? times and he went to house N for 2 times less
(a) 6 (b) 4 than to house M, then how many times R went
(c) 9 (d) 12 to house O in a month?
(a) 1 (b) 2
(e) None of these
(c) 3 (d) 4
10. Difference between speed of A and C is 20 (e) 5
km/h, find the time taken by A to travel 12. The ratio of the number of times potatoes
distance of 455 km, if his speed increased by seller P went to house N to O in a month was
10 km/h after every hour? 3:2, then how many fewer times did P go to
(a) 4.5 hours (b) 5.5 hours house O compared to N?
(c) 5 hours (d) 6 hours (a) 1 (b) 2
(e) Can’t be determined (c) 3 (d) 4
(e) 5
Direction (Q11-15): Study the following data
13. If S sold the potatoes to house N for only one
carefully and answer the questions:
time in a month and remaining potatoes were
The line graph represents the percentage of quantity
sold to house M and O, then quantity of
of potatoes (in kg) sold by 5 different vendors P, Q, R, potatoes sold to house O is how much
S and T in a month out of total potatoes sold by all the percentage of quantity of potatoes sold to
five sellers. house M and N together?
(a) 40% (b) 45%
% of potatoes sold (c) 50% (d) 55%
28%
30% (e) 60%
25% 22%
20%
20%
18% 14. The number of times Q went to house M and
15% 12% N in a month was x and y respectively, if x + y
10% = 11 and x - y = 5, then find the quantity of
5% potatoes sold by Q in house M?
0% (a) 24 kg (b) 30 kg
P Q R S T (c) 36 kg (d) 42 kg
(e) 48 kg
% of potatoes sold
15. Solve the following statement given below
The table graph shows the quantity of potatoes sold and mark the correct option.
at a particular time. The sellers went to at-least one Y = How many times did potatoes seller T go to
among 3 houses M, N and O to sell all quantity of house N in a given month?
potatoes. Z = If potatoes seller Q went only to house N in a
House Quantity of potatoes (in kg) sold month, then how many times did R go to house N
at a time in a given month?
M 6 (a) Y > Z (b) Z > Y
N 8 (c) Y = Z (d) Y ≥ Z
(e) Z ≥ Y
O 10

5
DAY-08
Direction (Q16-18) 19. 72.14% of 3499.89 – 79.76% of (1799.81 × ?
Read the following information carefully and answer ÷ 59.88) = 1599.92% of 119.86
the questions given below. (a) 120 (b) 60
M and N started a business with investment of Rs A (c) 80 (d) 25
and Rs A+400 for eight & ten months respectively. (e) 40
Profit share of N is Rs250 out of total profit of Rs400.
Cost price of an article is Rs5A and it marked up B%. 20. √(63.87 × 99.91 ÷ ?) + 239.80% of 299.90 =
25% discount allowed on the article and profit {24.85 × 20.18 + (14.89)2.2}
earned by 5%. (a) 196 (b) 225
16. A train can cross a ‘C’ meters long bridge in 16 (c) 144 (d) 289
seconds. The length of the train is ‘D’ meters (e) 256
𝑨
which is 3B< D < 𝟒. D is perfect square of the
21. (44.87% of 179.82) + (89.91% of 360.15) =
odd number which is multiple of five. If speed
(49.77% of 249.80) + (?% of 499.84)
of train is 25 m/sec, then find the time taken
(in seconds) by train to cover ‘2C’ platform. (a) 16 (b) 256
(a) 23 (b) 21 (c) 56 (d) 156
(c) 19 (d) 15 (e) 96
(e) 27
22. 15.012+19.912-586.86+156.33 of 24.98%=?
17. The average age of four (P, Q, R and S) (a) 55 (b) 57
students in a group is A/32. Age of P and Q is (c) 66 (d) 68
B years & 1.5B years respectively. If S is ten
(e) 77
years older than that of R, then find the
difference between ages of R and P (in years). Direction (Q23-26)
(a) A/60 (b) Both A and C
𝐴 In each of these questions a number series is given.
(c) B – 20 (d) + 10
𝐵 In each series only one number is wrong. Find out the
(e) Both A and D wrong number.
18. P alone and Q alone can complete a work in
23. 0.05, 3.6, 129.6, 2330.8, 20995.2, 94478.4
B/2 days and A/30 days respectively. P & R
together to work for seven days and they (a) 3.6 (b) 129.6
were replaced by Q. If Q alone complete the (c) 20995.2 (d) 2330.8
remaining work in five days and efficiency of (e) 0.05
R is ‘r’, then find which of the following
statement/s is /are true. 24. 28, 28, 35, 59, 124, 248, 463
(i) ‘r’ is multiple of two. (a) 35 (b) 248
(ii) ‘r+18’ is complete divisible by three. (c) 59 (d) 463
(iii) Efficiency of R is less than the efficiency of P. (e) 124
(a) Both (ii) and (iii) (b) Only (ii)
(c) None of these (d) Both (i) and (ii) 25. 82, 84, 96, 132, 212, 363, 614
(e) Only (i) (a) 84 (b) 363
(c) 96 (d) 614
Direction (Q19-22) (e) 132
What approximate value will come in place of the
question mark (?) in the following question? (Note: 26. 35, 91, 189, 343, 559
You are not expected to calculate the exact value.) (a) 559 (b) 35
(c) 189 (d) 91
(e) 343

6
DAY-08
Directions [Q27-29]: 31. Which of the following pair of symbols will
Study the data carefully and answer the following define the relation between Quantity I and
questions: Quantity II and between Quantity II and
Rohan, and Mohan, enter into partnership with Quantity III respectively?
initial investment of Rs. 37500, and Rs. 22500 (a) = (b) >
(c) < (d) ≥
respectively and after 6 months Sohan join them
(e) ≤
with initial investment of Rs. 45000. Further 6
F. # (relation cannot be established)
months later, both Rohan and Sohan again invested Quantity I: A path of uniform width of 6metres
Rs. 7500 more into the business. is to be made around a square park. If area of the
path is 1224 m2, then find the perimeter of the
27. If at the end of 2 years, profit earned by Sohan
park.
is Rs. 400 more than Mohan, then what is the
Quantity II: Train ‘M’ and ‘N’ are moving in the
total profit earned by them?
same direction. Train ‘M’ can cross train ‘N’ while
(a) Rs. 5400 (b) Rs. 2700 travelling in same direction in 21 seconds. If
(c) Rs. 8100 (d) Rs. 2160 speed of train ‘M’ is 126 km/h and speed of train
(e) Rs. 2400 ‘N’ is 54 km/h and train ‘N’ can cross a pole in 16
seconds, then find the length of train ‘M’.
28. If at the end of two years, total profit received
Quantity III: From a pizza in the shape of a circle
by them is Rs. 16200, then what is the profit of radius 28metres, a slice making an angle of 90o
earned by Rohan? at the centre is taken out. Find the perimeter of
(a) Rs. 2200 (b) Rs. 5500 the remaining pizza.
(c) Rs. 3300 (d) Rs. 6600 (a) A, B (b) A, C
(e) Rs. 1100 (c) A, A (d) B, A
(e) B, C
29. If at the end of 2 years, profit earned by Mojan
is Rs. 5400, then what is the difference 32. Quantity I: A three-digit number ‘xyz’ (> 100)
between the profit earned by Sohan and when digits are reversed it becomes ‘zyx’
such that zyx is more than ‘xyz by 198. Find all
Rohan?
possible values of ‘xyz?
(a) Rs. 900 (b) Rs. 800
Quantity II: There are two boats I and J rowing
(c) Rs. 1800 (d) Rs. 1200 in same river. Downstream speed of boat I is
(e) Rs. 1400 same as speed of boat in still water of boat J. Boat
I travels upstream distance of 280 km in 5 hours
Direction (Q30-34):
while boat J travels downstream distance of
In the following question, two statements numbered
1225 km in 12.5 hours. Find speed of boat in still
I and II are given. On solving them, we get quantities
water of boat I?
I and II, respectively. Solve for both the quantities (a) Quantity I > Quantity II
and choose the correct option. (b) Quantity I < Quantity II
(c) Quantity I ≥ Quantity II
30. If a² - b²= b (1 - a) - a (1 - b) such that a + b ≠
(d) Quantity I ≤ Quantity II
-1 and a and b are co-prime, then:
(e) Quantity I = Quantity II
Quantity I: Value of (a + b).
Quantity II: Value of (a2 + b2). 33. Article A and article B each sold at 20% profit,
(a) Quantity I ≥ Quantity II while marked price of A is 55% less than that
(b) Quantity I ≤ Quantity II of B. Article A sold after discount of (N – 10)
%, while B at (M + 25) % discount. Ratio of
(c) Quantity I > Quantity II
selling price of A and B is 3:5 respectively.
(d) Quantity I < Quantity II
Combined cost price of both articles is 3200,
(e) Quantity I = Quantity II and N is twice of M.

7
DAY-08
Quantity I: Find the value of N? 37. When the price of a mobile was reduced by
Quantity II: Find M% of 10% of Cost price of B? z% its sale increased by (2z) %. And
(a) Quantity I > Quantity II therefore net sale is increased by (21/2) %.
(b) Quantity I < Quantity II Which of the following option is true for the
(c) Quantity I ≥ Quantity II value of z?
(d) Quantity I ≤ Quantity II A. 35
(e) Quantity I = Quantity II B. 15
C. 10
34. A and B together can complete the work in 20
(a) Only A (b) Both A and C
days while B and C together can complete the
work in 15 days. A start the work and left (c) Both B and C (d) All three A, B and C
after 6 days and B joined the work and he also (e) Both A and B
left after 9 days. C completed the remaining
work in 13 days. Direction (Q38-39):
Quantity I: In how many days C alone can Following information gives the data regarding
complete the work? marks obtained by four students of a class in 1st and
Quantity II: In how many days A alone can 2nd monthly test.
complete the work? 1st monthly test: Marks obtained by D is 10 and 14
(a) Quantity I > Quantity II less than the marks obtained by A and B respectively.
(b) Quantity I ≤ Quantity II C and D together got 42 marks which is equal to the
(c) Quantity I < Quantity II marks obtained by A.
(d) Quantity I ≥ Quantity II 2nd monthly test: D got 10 marks more than B who
(e) Quantity I = Quantity II or relationship got 8 marks less than A. Half of marks obtained by C
cannot be established is 4 less than the marks obtained by A. Average of
marks obtained by A and D is 27.
35. Rs.12000 is divided between Roy, Joy, and
Sam. Share of Sam is equal to the 1/3rd of sum 38. Average marks obtained by A in both the
of share of Roy and Joy together. Share of Roy monthly tests are how much more or less
is equal to half of the share Joy, and Sam than the average marks obtained by D in both
together. Joy invests his share amount for 1.5 the monthly tests?
years at 20% per annum and the interest is (a) 4 (b) 8
being compounded half yearly. Find the total (c) 6 (d) 2
amount earned by Joy after 1.5 years? (e) 8
(a) Rs.6252 (b) Rs.6655 39. What is the ratio of marks obtained by A and
(c) Rs.6025 (d) Rs.6335 C together in 2nd monthly test to the marks
(e) None of these
obtained by C and B together in 1st monthly
36. P invested 50% of the amount invested by Q. test?
Q withdraw whole amount from the business (a) 3: 2 (b) 5: 4
after 4 months. R joins the business with (c) 2: 1 (d) 7: 6
investment of Rs. ____ in some month after Q (e) 4: 3
had withdrawn from the business. At the end
40. Pen having cost price of Rs. 550 is marked
of the year P and R share same amount of
44% above its cost price and sold after
profit. If investment of Q is Rs. 2400 then
offering a discount of 25%. Book was marked
which of the following may be the investment
Rs. 154 above its cost price and sold for Rs.
of the R.
556 after offering two successive discounts of
I. 1800
10% and Rs. 20 respectively. Find the ratio of
II. 3600
III. 2400 cost price of book to the selling price of pen
IV. 1600 respectively.
V. 7200 (a) 8:9 (b) 9:11
(a) i and ii (b) only iv (c) 6:5 (d) 7:9
(c) i, ii, iii and v (d) i, ii, iii and iv (e) 12:13
(e) i, ii and iv

8
DAY-08

Solutions
Solution (1-5) (7/5) = '140a'
1. Let runs scored by Rahul in match 2 be '100a'. And, runs scored by Kaif in match 2 = 100a ÷ 0.8
So, runs scored by Hardik in match 1 = 100a X = '125a'
(4/5) = '80a' Total runs scored by Rahul = 150a + 100a +
And runs scored by Hardik in match 3 = 100a X 200a = '450a'
(6/5) = '120a' And, total runs scored by Kaif = 60a + 140a +
Runs scored by Hardik in match 2 = 100a ÷ 1.25 125a = '325a'
= '80a' ATQ:
Runs scored by Rahul in match 1 = 100a X (3/2) 450a – 325a = 125
= '150a' Or, 125a = 125
And, runs scored by Rahul in match 3 = 100a X 2 So, a = 1
= '200a' So runs scored by Rahul in match 2 = 100x =
Runs scored by Kaif in match 1 = 100a X (3/5) = 100 X 1 = 100
'60a'
And, runs scored by Kaif in match 3 = 100a X
Similarly,
Runs
scored Total
Player Runs scored in match 1 Runs scored in match 2 in runs
match scored
3
Hardik 80 80 120 280
Rahul 150 100 200 450
Kaif 60 125 140 325

Total score of Delhi in match 2 = 80 + 100 + 125 = 305

2. Required difference = 200 - (80 + 60) = 60 If H = 2 then I = 5 (possible)


If H = 7 then I = 7 (possible) (Since value of H is≤
3. Required ratio = 150:(60 + 140) = 150:200 = 3:4
I)
Hence, option d.
So, possible value of (2I + 3H) = (2 x 5 + 3 x 2)
4. Runs scored by Delhi in match 3 = 120 + 200 + and (2 x 7 + 3 x 7) = 16 and 35
140 = 460 So, only I and III are possible
Runs scored by Mumbai batsmen = 460 - 79 - 31 Hence, answer is option C.
= 350
7.
5. Runs scored by Rahul in next match = 200 X K = 4,
0.75 = 150 So, points earned by B = (4 – 4) = 0
So, total score of the team in next match = 150 + 20 x I – 8H = 0
120 + 140 + 4 = 414 I/H = 2/5
So, required percentage = {(460 - 414)/460} X Minimum possible value of H = 5
100 = 10% So, G = 2 x 5 = 10
Hence, option a. Now,
6. Points scored by D = 20 x 8 – 8 x 3 = 136 20 x F – 8 x 10 > 0
Points scored by B = 136 – 52 = 84 F>4
Now, So, value of F = 5
20 x I – 8 x H = 84 Minimum possible points earned by C = 20 x 5 –
Number of I = (84 + 8 x H)/20 = (21 + 2 x H)/5 80 = 20
Hence, answer is option D.

9
DAY-08
8. Points earned by A = 20 x 5 – 8 x 3 = 76 So, 24x + 20x = 88
Points earned by C = 76 + 24 = 100 x=2
Now, ATQ, 3x – 2x = 3 * 2 – 2 * 2 = 2 times
20F – 8G = 100 13. Quantity of potatoes sold to house N = 8 kg
F = 5 + 2G/5 Quantity of potatoes sold to house M and O = 80
If G = 5, then F = 7 – 8 = 72 kg
So, minimum possible value of (F + G) = 7 + 5 = According to the notes, S went to house M was 4
12 times more than house O, so
Hence, answer is option A House M: House O = (x + 4): x
9. Points earned by D = 20 x 8 – 8 x 3 = 136 (x + 4) * 6 + x * 10 = 72
Now, 6x + 24 + 10x = 72
20I – 8H < 136 16x = 48
So, I < (136 + 8H)/20 x=3
If H = 3, then Z < 8 Quantity of potatoes sold in house M = 6 * 7 = 42
So, maximum possible value of I = 7, and kg
minimum possible value of H = 1 Quantity of potatoes sold in house O = 3 * 10 =
So, I + H = 7+1 = 8 30 kg
Hence, answer is option E. Required percentage = [30/ (42 + 8)] * 100 =
10. Ratio of time taken by A and C = 8:12 = 2:3 (30/50) * 100 = 60%
So, ratio of speed = 3:2 14. x + y = 11 -----eqn 1
So, speed of A = [3 / (3 – 2)] x 20 = 60 km/h x – y = 5 -------→ eqn 2
Distance travelled by A in 5 hours = 60 + 70 + 80 By solving 1 and 2,
+ 90 + 100 = 400 km. x = 8, y = 3
Rest distance travelled by him with speed of Quantity of potatoes sold by Q in house M = 6 * 8
110 km/h, so required time to cover the total = 48 kg
distance = 5 + 55/110 = 5.5 hours 15. For Y,
Hence, answer is option B Quantity of potatoes sold by T = 48 kg
11. According to the line graph, quantity of potatoes Quantity of potatoes sold at a time in house N =
sold by 8 kg
P = 400 * 22% = 88 kg According to the notes, T does not go to house M
Q = 400 * 18% = 72 kg and O.
R = 400 * 28% = 112 kg Then Y = 48/8 = 6 times
S = 400 * 20% = 80 kg For Z,
T = 400 * 12% = 48 kg Quantity of potatoes sold by R = 112 kg
Quantity of potatoes sold by R in Quantity of potatoes sold at a time in house N =
House M = 6 * 7 = 42 kg 8 kg
House N = 8 * (7 - 2) = 40 kg Then Z = 112/8 = 14 times
House O = 112 – (40 + 42) = 30 kg Hence, the correct answer is option b.
Required number of times from house O = 𝐴 ×8 150
16. (𝐴+400)×10 = 250
30/10 = 3 times
20A = 15 (A + 400)
12. Quantity of potatoes sold by P to house N in a
A = 1200
month = 8 * 3x = 24x
Cost price of the article = 5A = 5 × 1200 = Rs =
Quantity of potatoes sold by P to house O in a
6000
month = 10 * 2x = 20x 105
According to the notes, P does not go to house Selling price = 6000 × 100 = Rs.6300
M.

10
DAY-08
6300 59.88) = 1599.92% of 119.86
Marked price = 75
× 100 = Rs.8400
8400−6000 72% of 3500 – 80% of (1800 × ? ÷ 60) ~ 1600%
Markup % = × 100 = 40%
6000 of 120
B= 40 2520 – 0.80 × (30 × ?) ~ (16 × 120)
𝐴
Length of the train = 3B< D < 4 . 2520 – 1920 ~ 0.80 × (30 × ?)
= 120 < D < 300 (600/0.80) ~ (30 × ?)
D = 225 (225 is perfect square of 15 and 15 is (30 × ?) ~ 750
multiple of 5) ? ~ 25
ATQ 20. √(63.87 × 99.91 ÷ ?) + 239.80% of 299.90 =
225+𝐶
= 25 {24.85 × 20.18 + (14.89)2.2}
16
C = 175 √(64 × 100 ÷ ?) + 240% of 300 ~ {25 × 20 +
2 ×175+225 (15)2}
Required time = = 23 seconds
25 80 × (1/√?) + (2.4 × 300) ~ 500 + 225
17. Age of P = 40 years (80/√?) ~ 725 – 720
Age of Q = 1.5 × 40 = 60 years (80/√?) ~ 5
Let age of R be is s years √? ~ (80/5)
And age of S = s+10 √? ~ 16
1200
40 + 60 + s + ( s + 10 ) = × 4 ? = 256
32
110 + 2s = 37.5× 4 21. (44.87% of 179.82) + (89.91% of 360.15) =
s = 20 (49.77% of 249.80) + (?% of 499.84)
Required difference = 40 – 20 = 20 years (45% of 180) + (90% of 360) ~ (50% of 250) +
(?% of 500)
18. P alone can complete the work = 40/2 = 20 days
81 + 324 ~ 125 + (?% of 500)
Q alone can complete the work = 1200/30 = 40
405 – 125 ~ (5 × ?)
days
? ~ 280/5
Let total work = 40 units
? ~ 56
Efficiency of P and Q is 2 units and 1 units/day
Hence, option c.
respectively.
P & R together to work in seven days 7 × ( 2 +r 22. Answer is E.
) = (14 + 7r) units. 23. Wrong no is 2330.8
Amount of work done by Q in five days = 5 × 1 0.05 × 72 = 3.6
= 5 units 3.6 × 36 = 129.6
ATQ 129.6 × 18 = 2332.8
14 + 7r = 40 – 5 2332.8 × 9 = 20995.2
r=3 20995.2 × 4.5 = 94478.4
r is not multiple of 2
24. Wrong no is = 59
1st is false
28 + (13 - 1) = 28
From 2nd r = 3
28 + (23 - 1) = 35
r + 18 = 3 + 18 = 21
35 + (33 - 1) = 61
21 is divisible by 3
61 + (43 - 1) = 124
So, it is true
124 + (53 - 1) = 248
From 2nd Efficiency of R = units/ day
248 + (63 - 1) = 463
Efficiency of P = units/ day
It is false 25. Wrong no is = 363
So, answer is only 2 82 + ( 12 + 13 ) = 84
84 + ( 22 + 23 ) = 96
19. 72.14 % of 3499.89 – 79.76% of (1799.81 × ? ÷
96 + ( 32 + 33 ) = 132

11
DAY-08
132 + ( 42 + 43 ) = 212 So, (y + 240)/(35 – 15) = 21
212 + ( 52 + 53 ) = 362 Or, y + 240 = 420
362 + ( 62 + 63 ) = 614 Or, y = 180
26. The pattern is as follows: So, quantity II = 180 metres
23 + 33 = 35 Quantity III:
23 + 33 = 35 Desired perimeter = 3/4 × 2 × 22/7 × 28 + 28 ×
43 + 53 = 189 2 = 188 metres
53 + 63 = 341 So, Quantity III = 188 metres
63 + 73 = 559 Therefore, Quantity I = Quantity II < Quantity III
Hence, option b.
27. At the end of two years, the ratio of profit of
Rohan, Mohan, and Sohan = 37500 * 12 + 32. Quantity I,
(37500 + 7500) * 12 : 22500 * 24 : 45000 * 6 + Original number = 100x+ 10y + z
(45000 + 7500) * 12 = 990000: 540000: 900000 Number obtained by reversing the digits = 100z
= 11: 6: 10 + 10y + a
So, total profit = 400 * (11 + 6 + 10)/(10 - 6) = Now,
Rs. 2700 (100z + 10y + x) – (100x + 10y + z) = 198
(z – x) = 2
28. Profit earned by Rohan = 11/27 * 16200 = Rs.
Possible value of z = 3, 4…. 9 [7 values]
6600
Possible value of x = 1, 2…….7 [7 values]
29. Required difference = (11 - 10)/6 * 5400 = Rs. Possible values of y = 0, 1……... [10 values]
900 Required possible numbers = 7 x 10 = 70
30. a2 - b2 = b (1 - a) - a (1 - b) Quantity II,
a2 - b2 = b - ab - a + ab Let speed of boat in still water of boat I = a
(a - b) (a + b) = (b - a) km/h
(a - b) (a + b) + (a - b) = 0 Let speed of stream = b km/h
(a - b) (a + b + 1) = 0 Now,
Since a + b ≠ -1 which means a =b. (a – b) = 280/5 = 56…………... (1)
Let a = k and b = k (a + b + b) = (a + 2b) = 1225/12.5 = 98……… (2)
Since a and b are co-prime which means only On solving both equations, we get
value of a and b must be 1. 3b = 98 – 56
Value of (a + b) = 1 + 1 = 2 So, value of b = 42/3 = 14
Value of (a2 + b2) = 12 + 12 = 1 + 1 = 2 So, value of a (speed of boat in still water of boat
Hence, Quantity I = Quantity II I) = 56 + 14 = 70 km/h
So, Quantity I = Quantity II
31. Quantity I: Hence, answer is option E
Let length of side of the park is ‘z’ metres
33. Let cp of A and B is = A and B respectively
So, (z + 12)2 – z2 = 1224
Ratio of SP of A and B = 3:5
Or, 12 × (2z + 12) = 1224
Now,
Or, z + 6 = 51
1.2A/1.2B = 3/5
Or, z = 45
So, A/B = 3/5
So, perimeter of the park = 45 × 4 = 180 metres
So, Cost price of A = 3/8 x 3200 = 1200
So, Quantity I = 180 metres
CP of B = 5/8 x 3200 = 2000
Quantity II:
Ratio of MRP of A and B = 45:100 = 9:20
Speed of train ‘M’ = 126 × 5/18 = 35 m/s
And ratio of Selling price of A and B = 3:5
Speed of train ‘N’ = 54 × 5/18 = 15 m/s
Now,
Length of train ‘N’ = 15 × 16 = 240 metres
[9 x (100 – 2M + 10)]/ [20 x (100 – M – 25)] =
Let length of train ‘A’ is ‘y’ metres.

12
DAY-08
3/5 Option ii,
(110 – 2M)/ (75 – M) = 4/3 1200*12 = 3600*t
2M = 30 t = 4 months
So, M = 15 Option iii,
N = 30 1200*12 = 2400*t
Quantity I: Value of N = 30 t = 6 months
Quantity II: 15% x 10% x 2000 = 30 Option iv,
So, Quantity I = Quantity II 1200*12 = 1600*t
Hence, answer is option E t = 9 months (less than or equal to 8)
34. A and B together can complete the work in 20 Option v,
days 1200*12 = 7200*t
B and C together can complete the work in 15 t = 2 months
days Hence, the correct answer is option c.
According to the question, 37. Reduction in price = z%
6/A + 9/B + 13/C = 1 Increased in sale = 2z%
=> 6(1/A + 1/B) + 3(1/B + 1/C) + 10/C = 1 Net effect = (21/2) %
=> 6/20 + 3/15 + 10/C = 1 So,
=> 10/C = ½ (-z) + 2z + (-z)* (2z)/100 = 21/2
=> C = 20 days 100z – 2z2 = 2100/2
Quantity I: 2z2 – 100z + 1050 = 0
So, C alone can complete the work in 20 days z2 – 50z + 525 =0
Quantity II: (z - 35) (z - 15) = 0
B alone can complete the work in = 1/(1/15 - Either z = 35 or z = 15.
1/20) = 60 days Q(38-39)
So, A alone can complete the work in = 1/(1/20
38. 1st monthly test:
- 1/60) = 30 days
C and D together got 42 marks which is equal to
Hence, Quantity I < Quantity II
the marks obtained by A. Then,
35. Let share of Roy, Joy, and Sam is x,y and z Marks obtained by A = 42
respectively. Marks obtained by D is 10 and 14 less than the
So, x+y+z=12000 marks obtained by A and B respectively. Then,
So, 3z=x+y and 2x=y+z Marks obtained by D = 42 - 10 = 32
So, 3z+z=12000 Marks obtained by B = 32 + 14 = 46
Or, z=3000 And marks obtained by C = 42 - 32 = 10
2x-y=3000 and x+y=9000 2nd monthly test:
So, x= 12000/3=4000 and y = 9000-4000=5000 Let marks obtained by S = m
So, total amount earned after 1.5 years is S got 10 marks more than R who got 8 marks
= 5000(1+10/100)(1.5*2)=Rs.6655 less than P. Then,
36. Q invested amount and time period = 2400*4 Marks obtained by R = m - 10
P invested amount and time period = 1200*12 And marks obtained by P = m - 10 + 8 = m - 2
R invested amount and time period = x*t Average of marks obtained by A and D is 27.
ATQ, t = after Q withdrawn the business Then,
1200*12 = x*t (m + m - 2)/2 = 27
By using optional method, m - 1 = 27
Option i, m = 28
1200*12 = 1800*t Then, marks obtained by B = 28 - 10 = 18
t = 8 months Marks obtained by A = 28 - 2 = 26

13
DAY-08
Half of marks obtained by C is 4 less than the 40. Marked price of pen = 550 × 1.44 = Rs. 792
marks obtained by A. Then, Selling price of pen = 792 × 0.75 = Rs. 594
Marks obtained by C = 2 * (26 - 4) = 44 Let the cost price of book = Rs. Y
Average marks obtained by A in both the Then, marked price of book = Rs. (Y + 154)
monthly tests = (42 + 26)/2 = 34 Selling price of book = (Y + 154) × 0.9 – 20 =
And the average marks obtained by D in both 0.9Y + 138.6 – 20
the monthly tests = (32 + 28)/2 = 30 Or, (0.9Y + 118.6) = 556
Therefore, difference = 34 - 30 = 4 Or, Y = (556 – 118.6) ÷ 0.9 = 486
39. Marks obtained by A and C together in 2nd So, required ratio = 486:594 = 9:11
monthly test = 26 + 44 = 70 Hence, option b.
And the marks obtained by C and B together in
1st monthly test = 10 + 46 = 56
Therefore, ratio = 70: 56 = 5: 4

***

14
DAY-08

To enroll Rank File click here : Use Code Y179S to get maximum Discount

To enroll Bank Mahapack click here : Use Code Y179 to get maximum Discount

15

You might also like